2014 dxdy logo

Научный форум dxdy

Математика, Физика, Computer Science, Machine Learning, LaTeX, Механика и Техника, Химия,
Биология и Медицина, Экономика и Финансовая Математика, Гуманитарные науки




Начать новую тему Ответить на тему На страницу Пред.  1 ... 9, 10, 11, 12, 13, 14, 15 ... 27  След.
 
 
Сообщение14.07.2007, 15:36 


14/02/06
285
Два последних моих поста неверны - невнимательность - отвечал не на тот вопрос.

Прошу прощения, у меня полночь.

Теперь вижу ,что контрпример arcady работает.
Приношу Вам свои извинения за необоснованное ехидство.

 Профиль  
                  
 
 
Сообщение14.07.2007, 17:14 
Заслуженный участник


26/06/07
1929
Tel-aviv
Всё нормально, sergey1. Вы же были уверены в своей правоте.
А не ошибается тот, кто ничего не делает. Всё нормально!

 Профиль  
                  
 
 
Сообщение15.07.2007, 21:53 


15/03/07
128
Господин Артомонов, Вы рассуждаете, похоже, неверно.
В самом деле, Вы говорите, что $AB >= 9$
Нам необходимо показать, что $A >= \frac{1}{2}$.
Тогда нам необходимо (или достаточно - не знаю) показать, что
$B <= 18$ ( при этом $A >= \frac{9}{B} >= \frac{1}{2}$ ).
Но нер-во $B < 18$ не имеет места (более того, по-моему $B >= 18$ , причем
выражение $B$ принимает min, в силу наших ограничений, при $a=b=c=1$).

 Профиль  
                  
 
 
Сообщение15.07.2007, 22:02 


10/03/07
59
Казань
Можно несколько упростить задачу, если сделать её симметричной. Перепишем неравенство в виде:
$2[a(c^2+5)(a^2+5) + b(b^2+5)(a^2+5)+c(c^2+5)(a^2+5)]\geqslant(c^2+5) (a^2+5) (b^2+5) $
Применяя теорему К.-Б., усилим неравенство, заменяя левую часть на:
$2\sqrt { a^2+b^2+c^2} \sqrt{(c^2+5)^2 (a^2+5)^2+(b^2+5)^2(a^2+5)^2+ (c^2+5)^2(a^2+5)^2}$.
Применяя ко второму корню неравенство между средними, еще раз усилим левую часть:
$2\sqrt{3}\sqrt { a^2+b^2+c^2}(c^2+5)^{2/3} (a^2+5)^ {2/3}(b^2+5)^{2/3} \geqslant(c^2+5) (a^2+5) (b^2+5) $, или
$\frac{(c^2+5)^2 (a^2+5)^2 (b^2+5)^2}{ (a^2+b^2+c^2)^3} \leqslant 12^3$.
Судя по машинному графику, это усиленное неравенство также справедливо в заданной области. Хотя оно тоже не слишком простое, и доказать его я пока не успел.

Могу предложить в качестве побочного продукта занятные образцы, которые, однако, не принесли пользы для решения задачи.
$ab^2 + bc^2+ca^2\leqslant a^3+b^3+c^3$ для произвольных $a,b,c$, а также:
$a^2b^3 +b^2c^3 + c^2a^3\geqslant \frac{1}{3} (ab +bc +cd)^2$ для $\sum=3$.

 Профиль  
                  
 
 
Сообщение15.07.2007, 23:00 
Заслуженный участник


26/06/07
1929
Tel-aviv
Скорцонер писал(а):
Можно несколько упростить задачу, если сделать её симметричной. Перепишем неравенство в виде:
$2[a(c^2+5)(a^2+5) + b(b^2+5)(a^2+5)+c(c^2+5)(a^2+5)]\geqslant(c^2+5) (a^2+5) (b^2+5) $
Применяя теорему К.-Б., усилим неравенство, заменяя левую часть на:
$2\sqrt { a^2+b^2+c^2} \sqrt{(c^2+5)^2 (a^2+5)^2+(b^2+5)^2(a^2+5)^2+ (c^2+5)^2(a^2+5)^2}$.

Но в силу К-Б получаем:
$2\sqrt { a^2+b^2+c^2} \sqrt{(c^2+5)^2 (a^2+5)^2+(b^2+5)^2(a^2+5)^2+ (c^2+5)^2(a^2+5)^2}\geq$
$\geq2[a(c^2+5)(a^2+5) + b(b^2+5)(a^2+5)+c(c^2+5)(a^2+5)] .$
Согласно вашей терминологии, вы не усилили, а ослабили исходное неравенство.
То-бишь уже ничего нет. :wink:

Добавлено спустя 24 минуты 3 секунды:

Скорцонер писал(а):

Могу предложить в качестве побочного продукта занятные образцы, которые, однако, не принесли пользы для решения задачи.
$ab^2 + bc^2+ca^2\leqslant a^3+b^3+c^3$ для произвольных $a,b,c$, а также:
$a^2b^3 +b^2c^3 + c^2a^3\geqslant \frac{1}{3} (ab +bc +cd)^2$ для $\sum=3$.

Думаю, вы имеете в виду, что речь идёт о неотрицательных числах. :wink:
Первое можно доказать, например, так:
$ab^2 + bc^2+ca^2\leq a^3+b^3+c^3\Leftrightarrow\sum_{cyc}(a^3+2b^3-3ab^2)\geq0,$ что очевидно согласно AM-GM.
Второе - что-то интересное. Щас попробую.
Вы, конечно, имели в виду вот это:$a^2b^3 +b^2c^3 + c^2a^3\geqslant \frac{1}{3} (ab +bc +ca)^2$

Добавлено спустя 5 минут 40 секунд:

Да нет! Оно очевидно неверно при c=0. :wink:

 Профиль  
                  
 
 
Сообщение16.07.2007, 11:54 
Заслуженный участник
Аватара пользователя


07/03/06
1898
Москва
Pyphagor писал(а):
Господин Артомонов, Вы рассуждаете, похоже, неверно.
В самом деле, Вы говорите, что $AB >= 9$
Нам необходимо показать, что $A >= \frac{1}{2}$.
Тогда нам необходимо (или достаточно - не знаю) показать, что
$B <= 18$ ( при этом $A >= \frac{9}{B} >= \frac{1}{2}$ ).
Но нер-во $B < 18$ не имеет места (более того, по-моему $B >= 18$ , причем
выражение $B$ принимает min, в силу наших ограничений, при $a=b=c=1$).

Я рассуждал так.
Имеем $\frac{a}{b^2+5}+\frac{b}{c^2+5}+\frac{c}{a^2+5}=\frac{3}{5}+\frac{a^2c(a^2-5)+b^2a(b^2-5)+c^2b(c^2-5)}{5^3}+$ $\frac{a^6c(a^2-5)+b^6a(b^2-5)+c^6b(c^2-5)}{5^5}+...$. Ряд абсолютно сходится при $a,b,c<\sqrt{5}$, тогда каждый член правой части $\frac{a^2c(a^2-5)+b^2a(b^2-5)+c^2b(c^2-5)}{5^3}+$ $\frac{a^6c(a^2-5)+b^6a(b^2-5)+c^6b(c^2-5)}{5^5}+...$ отрицателен, и мы последовательно приближаемся к значению искомого выражения. Известно, что первые члены ряда играют определяющую роль в окончательном значении выражения. Поэтому, если мы хотим спуститься к $\frac 1 2$ или ниже, мы должны найти такие $$a,b,c$, при которых эти первые члены ряда принимают максимально отрицательное значение. Я учитываю два члена ряда, которые регулируют первый и второй знак. Остальные члены регулируют последующие знаки и как бы мы их не изменяли, если первые члены не минимальны, мы не спустимся ниже того, когда они минимальны, кроме возможно некоторой малой окрестности около точки, в которой первые члены минимальны. Поэтому решаем оптимизационную задачу $a^2c(a^2-5)+b^2a(b^2-5)+c^2b(c^2-5)\to\min\text{ при } a+b+c=3$ и находим $a=b=c=1$. Таким образом, мы доказали, что искомое неравенство будет верно при $a,b,c<\sqrt{5}$, кроме возможно некоторой окрестности точки $(1,1,1)$
Но, во-первых, это не решает полностью задачу, во-вторых, строго аналитически решить указанную оптимизационную задачу сложно (если и возможно), в третьих, такое решение сильно не по душе arqady, в четвертых, оно не нравится и мне.

 Профиль  
                  
 
 Re: Нестандартные задачи
Сообщение17.07.2007, 15:53 
Заслуженный участник


05/09/05
515
Украина, Киев
В этой теме одна задача рождает другую, а другая третью, так что я и не понял: решена ли одна из исходных задач?

student писал(а):

3)Докажите для положителные действительные числа $a,\ b,\ c$ справедливо
$\frac{a}{b+2c}+\frac{b}{c+2a}+\frac{c}{a+2b}\geq 1$



------
"Джентельмены, а поезд уже ушел?"
из к-ма "Человек с бульвара Капуцинов."

 Профиль  
                  
 
 Re: Нестандартные задачи
Сообщение17.07.2007, 17:42 
Заслуженный участник


26/06/07
1929
Tel-aviv
Macavity писал(а):
В этой теме одна задача рождает другую, а другая третью, так что я и не понял: решена ли одна из исходных задач?

student писал(а):

3)Докажите для положителные действительные числа $a,\ b,\ c$ справедливо
$\frac{a}{b+2c}+\frac{b}{c+2a}+\frac{c}{a+2b}\geq 1$



Эта задача очень лёгая и может быть решена многими способами, например, так:
$\frac{a}{b+2c}+\frac{b}{c+2a}+\frac{c}{a+2b}=\sum_{cyc}\frac{a^2}{ab+2ac}\geq\frac{(a+b+c)^2}{3(ab+ac+bc)}\geq1.$

 Профиль  
                  
 
 Re: Нестандартные задачи
Сообщение17.07.2007, 18:42 
Заслуженный участник


05/09/05
515
Украина, Киев
arqady писал(а):
Macavity писал(а):
В этой теме одна задача рождает другую, а другая третью, так что я и не понял: решена ли одна из исходных задач?

student писал(а):

3)Докажите для положителные действительные числа $a,\ b,\ c$ справедливо
$\frac{a}{b+2c}+\frac{b}{c+2a}+\frac{c}{a+2b}\geq 1$



Эта задача очень лёгая и может быть решена многими способами, например, так:
$\frac{a}{b+2c}+\frac{b}{c+2a}+\frac{c}{a+2b}=\sum_{cyc}\frac{a^2}{ab+2ac}\geq\frac{(a+b+c)^2}{3(ab+ac+bc)}\geq1.$


А может быть решение распространено на n-мерный случай?
И верно ли неравенство для n>3 (для n=2 и n=3 оно очевидно верно).

\sum_{i}\frac{a_i}{2a_{i+1mod(n)}+\sum_{j \neq i \wedge j \neq i+1mod(n) }{ a_j}} \geq1

Как-то так...

 Профиль  
                  
 
 
Сообщение17.07.2007, 21:04 
Заслуженный участник


26/06/07
1929
Tel-aviv
Macavity
Это верно по той же причине. Там всё сводится вот к такой радости: $$\sum_{cyc}(a_{i+1}-a_i)^2\geq0.$$
А вот это неравенство
$$\sum_{cyc}\frac{a_1}{a_2+2a_3+...+(n-1)a_n}\geq\frac{2}{n-1}$$
для положтельных $a_i$ и для натурального $n\geq2$ сводится к $$\sum_{sym}(a_i-a_j)^2\geq0.$$ :wink:

 Профиль  
                  
 
 
Сообщение18.07.2007, 13:23 
Заслуженный участник
Аватара пользователя


07/03/06
1898
Москва
Похоже верно следующее утверждение:
$\forall a,b,c\geqslant 0,a+b+c=3: \frac{a}{b^2+5}+\frac{b}{c^2+5}+\frac{c}{a^2+5}\geqslant \frac{2a}{b+11}+\frac{2b}{c+11}+\frac{2c}{a+11}$.
Тогда можно будет доказать неравенство:
$\frac{a}{b+11}+\frac{b}{c+11}+\frac{c}{a+11}\geqslant \frac{1}{4}$, а для него и метод неопределенных множителей Лагранжа не представляется таким уж неподъемным или разложением в ряд, как я уже показывал.
Вообще, при ограничениях на $a,b,c$ можно и дальше опускаться, но уже до тривиального $\frac{2a}{b+11}+\frac{2b}{c+11}+\frac{2c}{a+11}\geqslant \frac{a}{6}+\frac{b}{6}+\frac{c}{6}=\frac{1}{2}$, видимо можно и подниматься.

 Профиль  
                  
 
 
Сообщение18.07.2007, 17:05 
Заслуженный участник


26/06/07
1929
Tel-aviv
Артамонов Ю.Н. писал(а):
Похоже верно следующее утверждение:
$\forall a,b,c\geqslant 0,a+b+c=3: \frac{a}{b^2+5}+\frac{b}{c^2+5}+\frac{c}{a^2+5}\geqslant \frac{2a}{b+11}+\frac{2b}{c+11}+\frac{2c}{a+11}$.

По-моему, оно неверно для $c=0$ и $a=b=1.5$ :wink:

 Профиль  
                  
 
 
Сообщение18.07.2007, 19:15 
Заслуженный участник
Аватара пользователя


07/03/06
1898
Москва
Ну можно улучшать: берем $\frac{ak}{b+6k-1}+\frac{bk}{c+6k-1}+\frac{ck}{a+6k-1}$, где $k$ - некоторое число, например, $k=11$.

 Профиль  
                  
 
 
Сообщение18.07.2007, 21:21 
Заслуженный участник


26/06/07
1929
Tel-aviv
Артамонов Ю.Н.
Так выберите окончательный вариант и приведите полное доказательство.

 Профиль  
                  
 
 
Сообщение19.07.2007, 10:34 
Заслуженный участник
Аватара пользователя


07/03/06
1898
Москва
Приведу схему доказательства:
1. Нужно показать, что $\exists k_0 \forall k\geqslant k_0: \frac{a}{b^2+5}+\frac{b}{c^2+5}+\frac{c}{a^2+5}\geqslant \frac{ak}{b+6k-1}+\frac{bk}{c+6k-1}+\frac{ck}{a+6k-1}$;
2. $\forall k_1<k_2: \frac{ak_1}{b+6k_1-1}+\frac{bk_1}{c+6k_1-1}+\frac{ck_1}{a+6k_1-1}\geqslant \frac{ak_2}{b+6k_2-1}+\frac{bk_2}{c+6k_2-1}+\frac{ck_2}{a+6k_2-1}$.
Тогда очевидно, что $\forall k\geqslant 2: \frac{ak}{b+6k-1}+\frac{bk}{c+6k-1}+\frac{ck}{a+6k-1}\geqslant \frac 1 2 $, т.к. при $k\to\infty$ получим ровно $\frac 1 2$.
Осталось доказать п.1 и п. 2. :)

 Профиль  
                  
Показать сообщения за:  Поле сортировки  
Начать новую тему Ответить на тему  [ Сообщений: 401 ]  На страницу Пред.  1 ... 9, 10, 11, 12, 13, 14, 15 ... 27  След.

Модераторы: Модераторы Математики, Супермодераторы



Кто сейчас на конференции

Сейчас этот форум просматривают: нет зарегистрированных пользователей


Вы не можете начинать темы
Вы не можете отвечать на сообщения
Вы не можете редактировать свои сообщения
Вы не можете удалять свои сообщения
Вы не можете добавлять вложения

Найти:
Powered by phpBB © 2000, 2002, 2005, 2007 phpBB Group